Tamika is buying a handbag that is on sale for 20% off the original price. She uses a coupon to get an additional 5%
off the discounted price. If the original price of the handbag is $48.00, what is the final sale price that Tamika will pay?

Any help would be appreciated!

Answers

Answer 1

Answer:

The answer would be 90

Step-by-step explanation:

do 5 percent of 48


Related Questions

What is the solution to this equation?
X-9 = 17
O A. x = 28
O B. x = 12
O c. x = 8
O D. x = 26

Answers

Answer:

Step-by-step explanation:

x -  9 = 17

x = 26

the answer is D

solve for x (2x+1) (x+29)​

Answers

The answer would be 2x^2+59x+29

Please solve for a ! 24+5a=-a+6(5a-4)

Answers

Answer:

a = 2

Step-by-step explanation:

First of all you should remove parentheses by using distribution

24 + 5a = - a + 30a - 24

Then you should collect all like terms

24 + 5a = 29a - 24

After you should move the terms so the variables are only on one side.

5a - 29a = -24 - 24

Collect like terms

-24a = -48

After dividing both sides you get

a = 2

You borrow $350 from your aunt and agree to repay her $400 ($350 principal + $50 interest) in 18 months. What interest rate (using simple interest, and to the nearest tenth) are you paying?

Answers

Answer:

The interest rate is 0.79% per month.

Step-by-step explanation:

Present value of borrowed amount = $350

Future value of borrowed amount = $400

Interest amount = $50

Time duration = 18 months or 1.5 years

Now we have to find the rate of interest by using the above information. Here below is the calculation of interest rate.

Interest rate =  P×R×T

50 = 350 × R×18

R = 0.007936

Or, R = 0.79% per month.

help please will mark brainliest

Answers

Answer:

i can't see that

Step-by-step explanation:

plzz sent me a clear photo

Answer:

1. 1

2. 1  ⋅  10 ^24

3. 1000000000

4. 1

5. 1  ⋅  10 ^20

6. 1  ⋅  10 ^30

7. 1  ⋅  10 ^27

8. 10000

9. 0.01

10. 0.0001

11. 0.1

Hopefully these are right, I simplified all of the problems...sorry if they are wrong

Step-by-step explanation:

Let n be a whole number, and consider the statements below. p: n is a multiple of two. q: n is an even number. Which of the following is equivalent to ~q → ~p? A. ~q → ~p B. q → p C. p → q D. ~p →~

Answers

Answer:

C. p -> q

Step-by-step explanation:

Just did this on Edge2020. Hope this helps :)

The following data points represent the volume of gas in each race car driver's tank (in liters). Sort the data from least to greatest. Find the interquartile range (IQR) of the data set. \text{L}Lstart text, L, end text \text{L}Lstart text, L, end text

Answers

Answer:

its \text{L}Lstart text, L, end text

Step-by-step explanation:

The sorting of the dataset and the information from the five number summary, which includes the three quartiles and the minimum and maximum values indicates that we get;

1. 2.8, 4, 4.3, 6, 7.5, 8.5, 9, 11.6, 12, 12.1

2. The interquartile range is; 7.745

What is the five number summary of interquartile range of a data set?

The five number summary includes the first, second and third quartiles, Q₁, Q₂, and Q₃, and maximum and minimum values of the dataset, which serves to describe the characteristics of the data.

here, we have,

1. The sorted data from least to greatest can be presented as follows;

2.8, 4, 4.3, 6, 7.5, 8.5, 9, 11.6, 12, 12.1

Therefore, the original data in the question is already sorted from least to the greatest

2. The interquartile range, IQR, is the the difference between the third quartile, Q₃, and the first quartiles, Q₁, of the data.

IQR = Q₃ - Q₁

The dataset indicates that we get;

The number of datapoints in the dataset, n = 10

The first quartile = The (N + 1)/4 percentile value

First quartile, Q₁ = (10 + 1)/4 × 10 = The 2.75th value

The 2.75th value is the value 0.75 = 3/4 larger than the 2nd value, which can be found as follows;

Q₁ = The 2.75th value = 4 + (4.3 - 4) × (3/4)  = 4.225

The third quartile, Q₃ = The (N + 1) × (3/40th value, therefore;

Q₃ = The (10 + 1) × (3/4)th value = The 8.25th value (The value 0.25th more than the 8th value )

Q₃ = 11.6 + (12 - 11.6) × 0.25 = 11.7

The interquartile range, IQR = 11.7 - 4.225 = 7.475

The interquartile range = 7.475

Learn more on the interquartile range of a dataset here: brainly.com/question/13119591

#SPJ7

Complete question:

The following data points represent the volume of gas in each race car driver's tank (in liters).

Answer 2 questions about the data set.

1. Sort the data from least to greatest.

2.8

4

4.3

6

7.5

8.5

9

11.6

12

12.1

2. Find the interquartile range (IQR) of the data set.

L

Simple geometry equation

Answers

Answer:

106 or 4x+18  

Step-by-step explanation:

Since Ray QR is a bisector you can determine that PRQ and QRS are equal angles (def. of a bisector). Then, what I chose to do is graph them and find where they intercept which was 22. We now know 22 is x so we can substitute it into the equation and solve to deterine that our answer is 106.

22 = x, explanation:
x+ 31 = 3x-13
-x. -x
31=2x-13
+13.. +13
44-=2x
—. .—
2. 2
22= x

27x + 45y and 7(9x + 7y) are these equivalent?

Answers

Answer:

No

Step-by-step explanation:

If you want to know whether they are equivalent or no, we will first have to find the value of both.

[tex]27x+45y[/tex]

Since there aren't any like terms here, your answer would be

[tex]27x+45y[/tex]

Let's solve for next one.

[tex]7(9x+7y)[/tex]

Let's add in parenthesis to 7.

[tex](7)(9x+7y)[/tex]

Now we will separate everything. And also you will have to add.

[tex](7)(9x)+(7)(7y)[/tex]

Now that we added we got our answer.

[tex]63x+49y[/tex]

Both values were not the same, so therefore, your answer is no, they are not equivalent.

Hope this helps!

No

Step-by-step explanation:

If you want to know whether they are equivalent or no, we will first have to find the value of both.

Since there aren't any like terms here, your answer would be

Let's solve for next one.

Let's add in parenthesis to 7.

Now we will separate everything. And also you will have to add.

Now that we added we got our answer.

Both values were not the same, so therefore, your answer is no, they are not equivalent.

Hope this helps!

Read more on Brainly.com - https://brainly.com/question/17379473#readmore

Simplify the expression: z8 * z-3

Answers

Answer:

z^5

Step-by-step explanation:

z^8  * z^-3

Since we are multiplying exponents and the bases are the same, we can add the exponents

z^(8-3)

z^5

how do i solve this? xw+vy-xy-vw​

Answers

Here you go hopes this helps

Christine wants to buy strawberries and raspberries to bring to a party. Strawberries cost $1.65 per pound and raspberries cost $2.25 per pound. She only has $15 to spend on berries. Which inequality represents the situation where she buys x pounds of strawberries and y pounds of raspberries?

Answers

Answer:

1.65x + 2.25y ≤ 15

Step-by-step explanation:

Strawberries = $1.65 per pound Raspberries = $2.25 per pound

Amount with Christine = $15

Let

pounds of strawberries = x

pounds of raspberries = y

PxQx + PyQy ≤ 15

1.65x + 2.25y ≤ 15

The inequality which represent the situation where she buys x pounds of strawberries and y pounds of raspberries is 1.65x + 2.25y ≤ 15

1.65x + 2.25y less than or equal to 15

Identify the error in each problem. Explain each error and provide the correction solution. Use at least two complete sentences. PLEASE SOMEONE HELP QUICK ITS DUE TOMORROW AND I WILL MARK YOU BRAINLIEST

Answers

Answer:

the mistake is the addition sign. When a number is next to a parenthesis it means multiplication

The next model of a sports car will cost 12.9% more than the current model. The current model costs $35,000. How much will the price increase in dollars

Answers

Answer:

The price increase in dollars= $4,515

Step-by-step explanation:

Cost of the next model = cost of the current model + 12.9% cost of the current model

Cost of the current model = $35,000

12.9% cost of the current model

= 12.9/100 × $35,000

= 0.129 × $35,000

= $4,515

Cost of the next model = cost of the current model + 12.9% cost of the current model

= $35,000 + $4,515

= $39,515

Cost of the next model = $39,515

The price increase in dollars= next model price - current model price

=$39,515 - $35,000

= $4,515

The price increase in dollars = $4,515

i need help its urgent its due today geometry

Answers

Rays, vertex, the vertex of the angle,BXC

This year, Zachary has been babysitting his young cousins after school for $70 a month. His uncle also
gave him an extra bonus of $100 for his excellent work. Since school started, Zachary has earned more
than $500. How many months ago did school start?

Answers

Answer:

8

Step-by-step explanation:

One year, Mr James travels 8 x 10^3 miles for his job. The next year he traveled 1 x 10^4 miles. How many more miles did he travel the second year than he did the first year? Please give me the answer and explain ToT

Answers

Answer:

2 000 miles

Step-by-step explanation:

In the first year, Mr James travels 8 x [tex]10^{3}[/tex] miles = 8 000 miles.

The second year, he traveled 1 x [tex]10^{4}[/tex] miles = 10 000 miles.

Comparing the distance covered in the two years,

10 000 miles - 8 000 miles = 2 000 miles

Therefore, Mr James travels 2 000 more miles in the second year than he did the first year.

Let's assume that Mr James conveys himself to his place of work all through the first year in his private car. But in the second year, there were some days in which the car was not available. So he had to go for his job by public transportation. This itch during those days, could cause an increase in the distance covered from his home to his place of work for the second year.

What is the smallest positive number that is prime and 10 less than a perfect square?

Answers

Answer:

The problem states that the answer cannot be a perfect square or have prime factors less than $50$. Therefore, the answer will be the product of at least two different primes greater than $50$. The two smallest primes greater than $50$ are $53$ and $59$. Multiplying these two primes, we obtain the number $3127$, which is also the smallest number on the list of answer choices. So we are done, and the answer is $\boxed{\textbf{(A)}\ 3127}$.

Step-by-step explanation:

Answer:

71

Step-by-step explanation:

I just did the AOPS question, you can see the attachment down below.

Hope this helped! :)

I REALLY NEED HELP PLEASE HELP ME

Answers

Answer:

52^8

Step-by-step explanation:

26^8 = 208827064576 + 26

Jodi is considering taking online classes at two websites, LearnCenter and EduWorld. Each site requires that students pay a base membership fee, plus they charge a rate per class. The two graphs show the cost for taking online classes through the two websites. A graph titled Cost to Take Classes at LearnCenter has number of classes on the x-axis and cost in dollars on the y-axis. A line goes through points (0, 50) and (1, 70). A graph titled Cost to Take Classes at EduWorld has number of classes on the x-axis and cost in dollars on the y-axis. A line goes through points (0, 20) and (1, 60).

Answers

Answer:

B-LearnCenter charges a higher membership fee but a lower rate per class.

Step-by-step explanation:

Answer:B-learning for ever

Step-by-step explanation:

Which expression has the same value as -18-(-9)?
0 - 18+2
-12-(-3)
- 1945
O-8-(-4)

Answers

Answer:

the answer is -12-(-3)

Step-by-step explanation:

The expression equivalent to -18-(-9) is -12-(-3)

What are expressions?

An expression in maths is a sentence with a minimum of two numbers or variables and at least one maths operation.

Given is an expression, -18-(-9), we are given to find the expression equivalent to it,

-18-(-9) = -18+9 = -9

1) -18+2 = -16 (not equivalent)

2) -12-(-3) = -12+3 = -9

Therefore, we see, the value of expressions -12-(-3) and -18-(-9) are equivalent,

Hence, the expression equivalent to -18-(-9) is -12-(-3)

For more references on expressions, click;

https://brainly.com/question/14083225

#SPJ2


Need help on 3 thank you

Answers

Answer:

Cost: $10.00

Number of Tickets:26

Step-by-step explanation:

Each ticket is $0.50

20*$0.50=$10.00

For the tickets:

$13.00/$0.50=26 tickets

What is the measurement of this angle

Answers

Answer:

Acute, 40 degree angle

Step-by-step explanation:

Take 90 - 50 and you get 40

I hope this helps you :)

-KeairaDickson

Answer:

40°

Step-by-step explanation:

Count how many 10s it takes to get from 90 degrees to 50 and you get your answer...

meaning of cube root

Answers

Answer:

The cube root of a number is a special value that, when used in a multiplication three times, gives that number. Example: 3 × 3 × 3 = 27, so the cube root of 27 is 3. See: Square Root. Cubes and Cube Roots.

PLZ HELP ME ! Solve for k

Answers

Step-by-step explanation

if f(x)=4-3x*2 then f(-2) is

Answers

Answer:

16

Step-by-step explanation:

f(-2)=4-3(-2)*2

f(-2)=4-3(-4)

f(-2)=4+12

f(-2)=16

find 3+root 2/3-root 2=a+b root2

Answers

Answer:

a = [tex]\frac{11}{7}[/tex] ; b = [tex]\frac{6}{7}[/tex]

Step-by-step explanation:

[tex]\frac{3 + \sqrt{2}}{3 - \sqrt{2} } = a + b\sqrt{2} \\\\[/tex]

Rationalising [tex]\frac{3 + \sqrt{2}}{3 - \sqrt{2} }[/tex] gives :-

[tex]\frac{3 + \sqrt{2}}{3 - \sqrt{2} } = \frac{(3 + \sqrt{2})(3 + \sqrt{2})}{(3 - \sqrt{2})(3 + \sqrt{2}) } = \frac{(3 + \sqrt{2})^2}{3^2 - (\sqrt{2})^2 } = \frac{11 +6\sqrt{2} }{7}[/tex]

Comparing [tex]\frac{11 + 6\sqrt{2} }{7}[/tex] with [tex]a + b\sqrt{2}[/tex] gives

a = [tex]\frac{11}{7}[/tex] & b = [tex]\frac{6}{7}[/tex]

Answer:

[tex]\frac{11}{7}[/tex] + [tex]\frac{6}{7}[/tex] [tex]\sqrt{2}[/tex]

Step-by-step explanation:

Given

[tex]\frac{3+\sqrt{2} }{3-\sqrt{2} }[/tex]

Multiply the numerator/ denominator by the conjugate of the denominator.

The conjugate of 3 - [tex]\sqrt{2}[/tex] is 3 + [tex]\sqrt{2}[/tex] , thus

= [tex]\frac{(3+\sqrt{2})(3+\sqrt{2}) }{(3-\sqrt{2})(3+\sqrt{2}) }[/tex] ← expand numerator/ denominator using FOIL

= [tex]\frac{9+6\sqrt{2}+2 }{9-2}[/tex]

= [tex]\frac{11+6\sqrt{2} }{7}[/tex]

= [tex]\frac{11}{7}[/tex] + [tex]\frac{6}{7}[/tex] [tex]\sqrt{2}[/tex] ← in the form a + b[tex]\sqrt{2}[/tex]

with a = [tex]\frac{11}{7}[/tex] and b = [tex]\frac{6}{7}[/tex]

The area of a sector is 30 m2 in a circle with radius 4 m. What is the arc length of the sector?

Answers

Answer:

15 m

Step-by-step explanation:

The area of the circle is

A =pi r^2

A = pi 4^2 = 16 pi

The area of the sector is 30

The fraction is

30/16 pi  

Take this time 2pi which are the  radians of a circle

30 /16 pi * 2 pi = 15/4

This is the number of radians the angle is

The arc length is s = r * theta where theta is in radians

s = r  theta

  = 4 * 15/4

  = 15

Use the table to complete the work to find the missing value. Conversion Chart Pints Ounces 3 48 7 ? 30 480 3 pints 48 ounces 11 pints ? ounces How many ounces are in 11 pints? o 144 o160 o 176 o 192​

Answers

Answer:

176 ounces (C)

Step-by-step explanation:

if you divide 48 by 3 it will give you 16 and there are 11 pints so u wanna multiply 16 and 11 to 176 ounces..

also i got it right on edge :P

Answer: 176

Step-by-step explanation:

A scientist has two kinds of solutions. The first solution is Solution A, which is an $80\%$ acid solution. The second solution is Solution B, which is a $30\%$ acid solution. (a) Find the amount of Solution A (in mL) that must be added to $500$ mL of Solution B in order to produce a $70\%$ acid solution. (b) Find the amount of Solution A and Solution B (in mL) that can be combined in order to form a $100$ mL solution that is $50\%$ acid. (c) Does there exist a combination of Solution A and Solution B that is $90\%$ acid?

Answers

Answer:

Step-by-step explanation:

a ) Let the volume required be v .

v x .8 + 500 x .3 / ( v + 500 ) = .7

.8 v + 150 = .7 v + 350

.1 v = 200

v = 2000 mL

b )

Let the volume of solution A  required be v . The volume of solution of B required will be 100 - v

v x .8 + ( 100 - v ) x .3 / 100 = .5

.8 v + 30 - .3 v = 50

.5 v = 20

v = 40 mL

volume of solution B = 60 mL .

c )

It is not possible because 90 % is more than 80 % the concentration of solution A . Any mixture will have concentration less than 80 % .

volume of solution B = 60 mL

Other Questions
Why cant you taste or touch any materials at the Laboratory use this information for items 1-3. Aaron has $65 to rent a bike in the city . It costs $15 per hour to rent a bike. The additional fee for a helmet is $3 for the entire ride "Briefly explain ONE specific way that the societies of the colonies of South Carolina and Virginia differed AND why." Don't really need a specific answer, just a jumping-off point. what is the supply of labor? WIL GIVE BRAINLISEST PLEASE HELP SPEED ACCELERATION GRAPHS LABEL THE GRAPHS PLEASEEeeee How did the industrial revolution contribute to the spread of nationalism? Which of the following is not a valid use of your driver's license?proof of your ability to operate a motor vehicleO proof of your ageO proof of your residencyproof that you have liability insurance What is the base.... A line that passes through the points (2,1) and 6,9 PLEASE I WILL GIVE BRAINLIEST AND WORTH A TON OF POINTSSS Why does Frederick Douglass describe the distant relationship he had with his mother?to correct a misconception about his pastto demonstrate how well he recalls his formative yearsto criticize her for not protecting him from their masterto show how slavery can destroy the deepest of human bonds Compare and contrast personal development and personality development. Solve for t.Speed (Velocity)V = d/t find the equation perpendicular to the line y=2x-2 and passing through (-3 5) write a step by step proof. the givens are c is the midpoint of segment AB, and point c measures 10cm. prove AC=5cm plz help will mark brainliest Do you think most employers are serious about the development of their employees or are they only concerned with productivity? Draw a circle around which bond needs to bebroken to release energy.AdenineRibosephosphate The measure of an angle is 2. What is the measure of its complementary angle? what are the domain and range of the function represented by the set of order pairs? {(-12,-5),(-10,5),(10,-5),(12,5)} your alpaca is walking down the road at 4 m/s. how far does it go in 45 seconds? 3y^2+y^- 5+ 4y+2y^3+8